Intervertebral disc prolapse in the lumbar spine most often affects the L4/L5 and L5/S1 discs. In a man presenting with acute back pain following an episode of lifting a heavy weight, reduced force of which one of the following movements would most suggest an L4/L5rather than an L5/S1 disc lesion?

1. Ankle plantar flexion 2. Eversion of the foot 3. Extension of great toe 4. Inversion of the foot 5. Knee extension Explanation

Extension of great toe

In the lumbar spine (in contrast to the cervical spine) roots emerge below their respective vertebrae: so the majority of L4/5 disc prolapses would be expected to affect the L5 root and the majority of L5/S1 disc prolapses would affect the S1 nerve root. Although L5 contributes to hip abduction and extension, knee flexion and ankle dorsiflexion, weakness is often minimal because of the contribution of other roots to these movements and tends to be maximal in extension of the toes, particularly the great toe.

Ankle plantar flexion

Ankle plantar flexion is mediated by S1 and 2, with ankle dorsiflexion by L4/L5.

Eversion of the foot

Eversion of the foot is mediated by S1.

Inversion of the foot

Inversion of the foot is mediated by L4 alone.

Knee extension

Knee extension is mediated primarily by L3/4.

280 Rate this question: 1 2 3 4

A patient presents with pins and needles on the lateral and anterior aspect of his left thigh. On examination, there is no motor deficit. There is no history of trauma. Which of the following is likely to be causing the problem?

1. Lateral cutaneous nerve of the thigh lesion 2. L2 root lesion 3. L3 root lesion 4. Femoral nerve lesion 5. Saphenous nerve lesion Explanation

Lateral cutaneous nerve of the thigh lesion

The lateral cutaneous nerve of the thigh supplies the antero-lateral aspect of the thigh. It has no motor branches. is a condition in which there is irritation of the nerve causing sensory changes in the distribution of the lateral cutaneous nerve of the thigh without any motor changes.

L2 root lesion

L2 and L3 supply part of the dermatome described, but both have motor branches.

L3 root lesion

While the L3 nerve root supplies the dermatome, it also has motor branches, therefore if the L3 nerve root were affected weakness could be expected. The main muscle innervated by L3 is the quadriceps femoris.

Femoral nerve lesion

The femoral nerve supplies the quadriceps muscle therefore a lesion to the femoral nerve would result in weakness of knee extension.

Saphenous nerve lesion

The saphenous nerve is a terminal cutaneous branch of the femoral nerve. It runs with the saphenous vein to supply an area of skin below the knee on the medial aspect of the leg. Saphenous nerve can be seen after varicose vein surgery. Damage can also occur after vein harvest for coronary artery bypass graft (CABG).

297 Rate this question: 1 2 3

A patient presents with a history of back pain that developed 3 months ago when he got up suddenly from a seated position. The pain radiates down the leg to the ankle. On examination he has weakness of the quadriceps, reduced knee jerk reflex and reduced sensation over the patella. Where is the lesion likely to be?

1. compression 2. Ilioinguinal nerve 3. L3 nerve root 4. L5 nerve root 5. Compression of the femoral nerve at the inguinal ligament Explanation

L3 nerve root

The history suggests a prolapsed intervertebral disc. The quadriceps are supplied by the femoral nerve whose root value is L2–L4. The skin over the patella is usually part of the L3 dermatome, and the root value of the knee jerk is L3/L4.

Sciatic nerve compression

The sciatic nerve innervates the muscles of the posterior compartment of the thigh and the muscles of the leg. It provides sensory innervation for the posterior thigh, the leg and the foot.

Ilioinguinal nerve

The ilioinguinal nerve supplies a small area of skin on the medial aspect of the upper thigh as well as the scrotum and penis.

L5 nerve root

Compression of the L5/S1 nerve root produces symptoms of sciatica which would not result in a reduced knee jerk reflex.

Compression of the femoral nerve at the inguinal ligament

Femoral nerve compression at the level of the inguinal ligament is unlikely, given the history of injury and back pain.

339 Rate this question: 1

A patient complains of sensory deficit halfway down the anterior surface of the thigh. The patient is very disconcerted by their presentation. Which one of the following would cause this sensory deficit?

1. Damage to the sciatic nerve 2. Compression of the ventral roots of L5 to S2 3. Damage to a nerve accompanying the artery in the adductor canal 4. Could be the result of nerve damage during surgical procedures in the femoral sheath 5. Damage to the nerve that innervates the pectineus muscle Explanation

Damage to the nerve that innervates the pectineus muscle

The anterior surface of the thigh receives its sensory innervation from the femoral nerve so this is the nerve most likely to be injured. The pectineus is supplied by the second, third and fourth lumbar through the femoral nerve. Occasionally, it receives a branch from the obturator nerve.

Damage to the sciatic nerve

This would result in sciatica in which pain is experienced in the lower back with extension of the pain down the posterior thigh.

Compression of the ventral roots of L5 to S2

Compression of the L5–S2 nerve root would result in posterior sciatica symptoms.

Damage to a nerve accompanying the artery in the adductor canal

The saphenous nerve (a branch of the femoral nerve) can be found in the adductor canal. It has no motor function but provides sensory supply to the medial aspect of the lower half of the leg.

Could be the result of nerve damage during surgical procedures in the femoral sheath

The femoral branch of the genitofemoral nerve can be damaged within the femoral sheath. This supplies skin around the femoral triangle area.

367 Rate this question: 1 2 3

A young man sustains a skull-base fracture at the middle cranial fossa that injures his right abducens (VI) nerve. Which signs are most likely to be present on clinical examination?

1. There is ptosis on the right side 2. The pupil on the right side is constricted and fails to respond to light 3. The right eyelid is numb 4. The patient is unable to deviate his right eye medially 5. The patient is unable to deviate his right eye laterally

Explanation

The patient is unable to deviate his right eye laterally

The abducens nerve innervates the lateral rectus muscle of the eye exclusively; the sole effect of damage to this nerve is that the patient is unable to abduct (laterally deviate) the eye.

There is ptosis on the right side

Ptosis would be as a result of damage to the oculomotor nerve.

The pupil on the right side is constricted and fails to respond to light

The pupillary light reflex is detected by the optic nerve and pupillary constriction is initiated by the oculomotor nerve.

The right eyelid is numb

The trigeminal nerve is responsible suppling facial sensation including to the right eyelid.

The patient is unable to deviate his right eye medially

The oculomotor nerve via the medial rectus muscle is responsible for adduction of the eye.

371 Rate this question: 1 2

In performing a lumbar puncture, the operator needs to be familiar with the anatomy involved. There are important structures in the area that can be inadvertently damaged. Which anatomical feature is relevant to this procedure?

1. In the newborn baby, the spinal cord occupies the full length of the dural sac 2. The dural sac in the adult terminates at the lower end of the sacral canal 3. The spinal cord in the normal adult terminates anywhere from opposite the body of T12 to the body of L3; however, the commonest level is at the disc space between L1 and L2 4. The spinal cord in the average male is 12 inches (30 cm) in length 5. The extradural space comprises a thin layer of avascular connective tissue Explanation

The spinal cord in the normal adult terminates anywhere from opposite the body of T12 to the body of L3; however, the commonest level is at the disc space between L1 and L2

During a lumbar puncture, a needle most commonly passes through the L3/4 intervertebral space (usually found at the height of the iliac crest) to avoid the spinal cord.

In the newborn baby, the spinal cord occupies the full length of the dural sac

In the newborn, it terminates at L3.

The dural sac in the adult terminates at the lower end of the sacral canal

The dural sac in the adult extends to the level of the second sacral segment.

The spinal cord in the average male is 12 inches (30 cm) in length

The spinal cord in both adult men and women is 18 inches (45 cm) in length.

The extradural space comprises a thin layer of avascular connective tissue

The spinal extradural space contains loose fat (which allows the ready diffusion of local anaesthetic in an extradural block), together with the extensive vertebral venous plexus of veins.

372 Rate this question: 1 2 3 4 5

A cerebral angiogram is performed on a 37-year-old woman, following a suspected aneurysmal bleed. She presented with reduced GCS and headache without a history of trauma. Which anatomical feature should be considered when interpreting the angiogram?

1. The middle cerebral artery is the largest artery among the cerebral arteries 2. The posterior cerebral artery is clearly seen on a lateral carotid angiogram 3. The vertebral arteries meet at the foramen magnum to form the basilar artery 4. The middle cerebral artery courses over the lateral aspect of the temporal lobe of the cerebrum 5. The middle meningeal artery is an extracranial branch of the internal carotid artery Explanation

The middle cerebral artery is the largest artery among the cerebral arteries

The middle cerebral artery is indeed the largest artery among the cerebral arteries, being, in effect, the termination of the internal carotid.

The posterior cerebral artery is clearly seen on a lateral carotid angiogram

The posterior cerebral artery arises from the termination of the basilar artery, which itself arises from the two vertebral arteries that meet on the under surface of the brainstem – so the posterior cerebral artery can only be visualised by vertebral angiography.

The vertebral arteries meet at the foramen magnum to form the basilar artery

The vertebral arteries meet under the surface of the brainstem.

The middle cerebral artery courses over the lateral aspect of the temporal lobe of the cerebrum

It passes through the lateral sulcus of the cerebrum between the temporal and frontal lobes.

The middle meningeal artery is an extracranial branch of the internal carotid artery

The middle meningeal artery is a branch of the maxillary artery, one of the terminal branches of the external carotid.

373 Rate this question: 1 2 3 In clinic, you see a 36-year-old man referred by his general practitioner (GP) with a feeling of numbness in his left leg. On examination, he has decreased position sense and light touchand vibration sensation affecting his left leg to the upper part of the thigh. No other neurological deficit is demonstrable. Which one of the following is the most likely cause of this presentation?

1. Left dorsal column lesion 2. Left spinothalamic tract lesion 3. Peripheral 4. Partial section of the spinal cord 5. Sensory root compression Explanation

Left dorsal column lesion

The dorsal columns carry ipsilateral proprioception, light touch and vibration sensation.

Left spinothalamic tract lesion

Spinothalamic tract lesions cause contralateral loss of pain and temperature sensation.

Peripheral polyneuropathy

Peripheral polyneuropathy is usually bilateral.

Partial section of the spinal cord

Partial section of the cord tends to cause contralateral pain and temperature sensation and ipsilateral loss of the modalities carried in the dorsal columns; there may also be upper motor neurone weakness. A clinical example is Brown–Séquard syndrome, caused by damage to one-half of the spinal cord.

Sensory root compression

Sensory root compression typically causes pain in the dermatome supplied by the root.

375 Rate this question: 1 2 3 4 5

A patient has been diagnosed with a fast-growing pituitary adenoma. Magnetic resonance image (MRI) scanning reveals suprasellar extension. Which structure is most likely affected?

1. Abducens nerve 2. Hypothalamus 3. Oculomotor nerve 4. Third ventricle 5. Optic chiasm Explanation

Optic chiasm

The pituitary gland occupies the sella turcica, which is a cup-shaped depression in the basisphenoid bone. The roof of the sella is formed by the diaphragma sella, a fold of dura, which is perforated to allow passage of the pituitary stalk. Above the diaphragm lies the suprasellar cistern, the optic chiasm and the anterior cerebral arteries. The optic chiasm is most likely to be compressed by a pituitary adenoma. Classically, this produces a bitemporal hemianopia.

Abducens nerve

The lateral walls of the pituitary fossa are formed by the cavernous sinuses which contain the internal carotid arteries and cranial nerves III, IV, V1, V2 and VI. The abducens nerve is less likely to be compressed than the optic nerve as adenomas extend upwards.

Hypothalamus

The hypothalamus is located superiorly to the pituitary gland and the optic chiasm within the brain parenchyma. With a rapidly growing adenoma, the optic nerve becomes compressed between the hypothalamus and the adenoma.

Oculomotor nerve

The oculomotor nerve forms part of the lateral wall of the pituitary fossa; however, as adenomas usually extend upwards, it is less likely to be affected than the optic chiasm.

Third ventricle

Even if the third ventricle were compressed by a rapidly growing adenoma, there is unlikely to be any clinical significance.

377 Rate this question: 1 2 3 You review a 56-year-old woman in the Outpatient Clinic following myocardial revascularisation using the internal thoracic arteries. Despite a strong recovery and good postoperative course, she complains that since the operation she has had chronic pain in her shoulder and that her shoulder is becoming weaker. When you examine her, you notice winging of the scapula. What is the most likely cause of these symptoms?

1. Damage to the axillary nerve 2. Damage to the 3. Damage to the 4. Damage to the spinal accessory nerve 5. Damage to the whole Explanation Damage to the long thoracic nerve Damage to the long thoracic nerve causes winging of the scapula due to weakness of the serratus anterior.

Damage to the axillary nerve

The axillary nerve contains fibres of C5 and C6 to supply predominantly the deltoid muscle. It conveys some sensory fibres to the lateral aspect of the forearm. Damage to the radial nerve The radial nerve supplies the extensor carpi radialis and ulnaris, plus extensor digitorum (via the posterior interosseous nerve), so its injury results in . Damage to the spinal accessory nerve Spinal accessory nerve damage will cause weakness in shrugging the shoulders. Damage to the whole brachial plexus

Damage to the whole brachial plexus would produce a wide range of symptoms including significant motor and sensory deficits.

394 Rate this question:

1 2 3 4 5

A 27-year-old man has collapsed following a subarachnoid haemorrhage. After initial resuscitation and investigations, he is taken to theatre and the aneurysm is clipped. After waking from anaesthesia his Glasgow Coma Score (GCS) is 15/15 and he is taken to the Neurological Intensive Care Unit (ITU) for monitoring. When you see him on the ward round the next day his GCS has fallen to 12/15 and a computed tomography (CT) scan shows dilated ventricles. Which one of the following is correct about cerebrospinal fluid (CSF) circulation?

1. It flows between the third and fourth ventricles via the aqueduct of Monro 2. It is produced at a rate of 2 ml/min 3. It is produced by the arachnoid granulations 4. It is sterile 5. It normally has a higher protein content than plasma Explanation It is sterile Cerebrospinal fluid is usually sterile. It is only non-sterile in the context of infection, for example meningitis.

It flows between the third and fourth ventricles via the aqueduct of Monro

The aqueduct of Sylvius connects the third and fourth ventricles; the foramen of Monro connects the lateral ventricles. It is produced at a rate of 2 ml/min CSF is produced at a rate of 0.3–0.5 ml/min.

It is produced by the arachnoid granulations

The arachnoid granulations (villi) are involved in the reabsorption of the CSF into the venous sinuses and so into the systemic circulation. Cerebrospinal fluid is predominantly produced by the choroid plexuses in the ventricles. It normally has a higher protein content than plasma The protein content of serum is much higher than that of the CSF.

395 Rate this question: 1 2 3 4 5

A 36-year-old man presents with sensory loss of his right medial leg distal to the knee. On physical exam, he is unable to extend his right knee and his patellar reflex is absent. Which would produce these symptoms? 1. A common peroneal nerve injury 2. A lesion of L3 nerve root 3. A lesion of S1 4. L4 nerve root compression 5. Sciatic nerve injury Explanation L4 nerve root compression Lesions affecting the L3/4 nerve root also results in a decreased knee reflex, and sensory loss over the anterior thigh and medial part of the leg. In this particular scenario, it is also more likely that is the L4 root, rather than the L3 root, that is affected due to the pattern of sensory loss described. A common peroneal nerve injury Common peroneal (or fibular) nerve injuries often occur from pressure from plaster casts and lateral knee ligament injuries. Superficial peroneal nerve injuries impair eversion but not dorsiflexion, as this action is subserved by the deep peroneal nerve. Peroneal nerve injury does not cause the pattern of injury described here. A lesion of L3 nerve root The L3 nerve root supplies a dermatomal area across the mid-thigh and extends to just beyond the knee. Given the sensory deficit is distal to the knee, it is the L4 nerve root that is more likely affected. A lesion of S1 S1 lesions result in a weak plantar flexion, loss of ankle reflex and sensory loss over the sole of the foot. Sciatic nerve injury Sciatic nerve injury, which can occur through trauma during surgery or with gluteal intramuscular injection, results in sensory deficit on the posterior aspect of the thigh and does not extend beyond the knee. It can also present with foot drop.

400 Rate this question: 1 2 3 4 5

A 69-year-old man falls sustaining a hyperextension injury to his neck. On examination, he has weakness in his upper and lower limbs, but sensation is preserved. Which spinal tract injury explains this presentation? 1. The dorsal column 2. The lateral spinothalamic tract 3. The ventral spinothalamic tract 4. The lateral corticospinal tract 5. The ventral corticospinal tract Explanation The lateral corticospinal tract The lateral corticospinal tract transmits voluntary motor signals to the limbs. The dorsal column The dorsal columns are white matter tracts formed by the fasciculus gracilis and fasciculus cuneatus. The fasciculus gracilis lies medially to the fasciculus cuneatus. They carry fine touch and proprioception. The lateral spinothalamic tract The lateral spinothalamic tracts carry crossed axons conducting pain and temperature sensations. The ventral spinothalamic tract The ventral (or anterior) spinothalamic tract contains ascending fibres that transmit crude touch and firm pressure. The ventral corticospinal tract The ventral (or anterior) corticospinal tract contains descending fibres and control axial voluntary muscles. It is much smaller than the lateral corticospinal tract and decussate at the level of the spinal cord.

423 Rate this question: 1 2 3 4 5

A young man sustains following a stabbing incident. He presents with weakness of the elbow extension and wrist dorsiflexion. Which part of the brachial plexus has been transected? 1. C 5 nerve root 2. The superior trunk 3. The posterior cord 4. The axillary nerve 5. The Explanation The posterior cord The posterior cord of the brachial plexus gives off the radial nerve, which supplies the triceps and the extensor carpi radialis. The brachial plexus is derived from the anterior rami of C5, 6, 7, 8, T1. It is described in terms of roots, trunks, divisions, cords and nerves. The musculocutaneous nerve (C5, 6, 7) is a continuation of the lateral cord, the (C5, 6, 7, 8, T1) is derived from the medial and lateral cords, the ulnar (C8, T1) nerve is a continuation of the medial cord, and the radial nerve (C5, 6, 7, 8, T1) a continuation of the posterior cord. C 5 nerve root The C5 nerve supplies nerves to the brachial plexus, however transection of the C5 nerve root would result in weakness of shoulder abduction and external rotation. The superior trunk The superior (or upper) trunk is derived from the C5/6 nerve roots. Damage to this area would result in limited elbow flexion, medial rotation of the arm, forearm pronation, and loss of sensation over the deltoid and the lateral . The axillary nerve The axillary nerve is not involved in elbow extension and wrist dorsiflexion. The ulnar nerve The ulnar nerve is involved in innervation of the forearm and hand and, therefore, would not be involved in elbow extension or wrist dorsiflexion.

424 Rate this question: 1 2 3 4 5

An elderly man with pre-existing cervical spondylosis falls sustaining hyperextension injury to his neck. On examination he has motor deficit that is worse in the upper limb than the lower limb. Which spinal cord injury explains this presentation? 1. Complete spinal cord injury 2. Anterior cord syndrome 3. Brown-Séquard syndrome 4. Central cord syndrome 5. Posterior cord syndrome Explanation Central cord syndrome Central cord syndrome typically occurs following cervical hyperextension injury, in which anterior and posterior cord compression can lead to oedema, haemorrhage or ischaemia. It is thought to be a result of vascular compromise of the cord in the distribution of the anterior spinal artery. Infarction of the cord in the territory of this artery could also result in anterior cord syndrome. Central cord syndrome is characterised by a disproportionately greater loss of motor power in the upper extremities than in the lower extremities, with varying degrees of sensory loss. The arms and hands are most severely affected as the motor fibres to the cervical segments are topographically arranged towards the centre of the cord. Complete spinal cord injury In contrast with all the above, a complete spinal cord injury would feature a complete loss of sensory and motor function below the level of lesion. Anterior cord syndrome Anterior cord syndrome is generally caused by ischaemic injury to the anterior spinal injury and is characterised by paraplegia below the level of the lesion (due to corticospinal tract involvement) and a dissociated sensory loss with loss of pain and temperature sensation (due to lateral spinothalamic tract involvement). Position, vibration and deep pressure sensations, all functions of the posterior column, are preserved. Anterior cord syndrome has the poorest prognosis of all incomplete spinal injuries.

Brown-Séquard syndrome

Brown–Séquard syndrome, resulting from hemisection of the cord, usually causes ipsilateral motor loss and contralateral loss of pain and temperature sensation; there is also associated loss of position sense. Posterior cord syndrome Posterior cord syndrome is a rare, incomplete lesion with primary damage to the posterior cord, featuring preservation of motor function, pain and temperature sensations with loss of proprioception and fine touch sensation below the level of the lesion. It can occur secondarily to a posterior spinal artery infarct.

426 Rate this question: The patellar reflex is a monosynaptic deep tendon reflex arc that helps maintain posture and balance. Normally it can be easily elicited with a tendon hammer. An absent knee reflex is most likely to be due to which one of the following? 1. Dorsal column demyelination 2. Femoral nerve transection 3. L1–L2 disc prolapse 4. S2–S4 dorsal nerve root avulsions 5. Old T12 level cord transection Explanation Femoral nerve transection The patellar reflex is mediated by the femoral nerve formed from the posterior divisions of the L2–L4 anterior spinal rami and is, therefore, lost after femoral nerve and L2–L4 dorsal root damage. Dorsal column demyelination Dorsal column lesions only affect central sensory processing as collaterals subserving spinal reflexes are preserved. L1–L2 disc prolapse L1-L2 disc prolapse is too high to cause absent patellar reflex. With L2 prolapse the reflex may be slightly diminished, but should not be absent. S2–S4 dorsal nerve root avulsions The patellar reflex synapses in the L2–4 anterior spinal rami, and as such the S2–4 nerve roots, are too low to affect the patellar reflex. Furthermore, the anterior nerve roots would need to be affected, not the dorsal (or posterior) nerve roots. Old T12 level cord transection Although an acute T12 level cord transection may result in a transient loss of reflexes, T12 cord lesions typically result in an upper motor neurone lesion with exaggerated reflexes.

430 Rate this question: 1 2 3 4 5

A 78-year-old man had poliomyelitis as a child that left him with total paralysis of the left deltoid muscle. Which feature is most likely to be present on clinical examination?

1. Anaesthesia over the ‘epaulette’ region of the left shoulder 2. The acromion process of the scapula forms the most lateral bony landmark of the left shoulder 3. Drooping of the left shoulder compared to the right side 4. Detectable weakness flexing the shoulder and internally rotating the shoulder when this is compared with the right side 5. Full abduction of the shoulder can be achieved by the action of the intact supraspinatus muscle on the left side Explanation

Detectable weakness flexing the shoulder and internally rotating the shoulder when this is compared with the right side

The deltoid is the primary abductor of the arm. Even if the supraspinatus is fully functional, it is far too weak a muscle to be able to abduct the whole weight of the arm. The deltoid, in addition to being the powerful abductor of the humerus, also assists in flexion/extension and medial/lateral rotation of the shoulder. Weakness of these movements compared with the normal side can be detected on careful examination.

Anaesthesia over the ‘epaulette’ region of the left shoulder

This would be as a result of damage to the axillary nerve.

The acromion process of the scapula forms the most lateral bony landmark of the left shoulder

The most lateral bony landmark of the shoulder is the greater tubercle of the head of the humerus.

Drooping of the left shoulder compared to the right side

Drooping of the shoulder would be due to damage to the accessory nerve, causing weakness in the trapezius muscle.

Full abduction of the shoulder can be achieved by the action of the intact supraspinatus muscle on the left side

The supraspinatus muscle is far too weak to fully abduct the whole arm. It can initiate the first 10–15° of abduction.

462 Rate this question: 1 2 A patient attends the clinic complaining of pins and needles affecting the radial (lateral) three digits of his hand. The symptoms are worse at night and also after driving long distances. Which one of the following motor signs would confirm the likely diagnosis?

1. Inability to flex the radial three fingers 2. Inability to adduct all the fingers of his hand 3. Inability to abduct all the fingers of his hand 4. Weakness of opposition of the thumb 5. The likely diagnosis would not cause a motor deficit Explanation

Weakness of opposition of the thumb

The history described above is typical of syndrome. Entrapment of the median nerve at the carpal tunnel affects the muscles of the . These are the abductor pollicis brevis, flexor pollicis brevis and opponens pollicis. The nerve supply of the flexor pollicis brevis is extremely variable however, so the best test to perform is assessment of opposition. In addition, the motor branch of the median nerve after the level of the carpal tunnel also innervates the radial two lumbricals.

Inability to flex the radial three fingers

Flexion of the radial three fingers would not be affected as the nerve supply to the long flexors comes from the median nerve proximal to the carpal tunnel. The lateral terminal branch of the median nerve (which passes through the carpal tunnel) innervates the flexor pollicis brevis, however this is responsible for flexion of the thumb, whereas the question refers to flexion of the fingers.

Inability to adduct all the fingers of his hand

Abduction and adduction of the fingers is effected by the interossei, which are supplied by the ulnar nerve.

Inability to abduct all the fingers of his hand

Abduction of the fingers is effected by the interossei, which are supplied by the ulnar nerve.

The likely diagnosis would not cause a motor deficit

The median nerve supplies muscles within the thenar eminence including the abductor pollicis brevis, flexor pollicis brevis and opponens pollicis and so there would be a motor deficit.

494 Rate this question: 1 A patient attends the clinic complaining of pins and needles affecting the radial (lateral) three digits of his hand. The symptoms are worse at night and also after driving long distances. Which one of the following motor signs would confirm the likely diagnosis?

1. Inability to flex the radial three fingers 2. Inability to adduct all the fingers of his hand 3. Inability to abduct all the fingers of his hand 4. Weakness of opposition of the thumb 5. The likely diagnosis would not cause a motor deficit Explanation

Weakness of opposition of the thumb

The history described above is typical of . Entrapment of the median nerve at the carpal tunnel affects the muscles of the thenar eminence. These are the abductor pollicis brevis, flexor pollicis brevis and opponens pollicis. The nerve supply of the flexor pollicis brevis is extremely variable however, so the best test to perform is assessment of opposition. In addition, the motor branch of the median nerve after the level of the carpal tunnel also innervates the radial two lumbricals.

Inability to flex the radial three fingers

Flexion of the radial three fingers would not be affected as the nerve supply to the long flexors comes from the median nerve proximal to the carpal tunnel. The lateral terminal branch of the median nerve (which passes through the carpal tunnel) innervates the flexor pollicis brevis, however this is responsible for flexion of the thumb, whereas the question refers to flexion of the fingers.

Inability to adduct all the fingers of his hand

Abduction and adduction of the fingers is effected by the interossei, which are supplied by the ulnar nerve.

Inability to abduct all the fingers of his hand

Abduction of the fingers is effected by the interossei, which are supplied by the ulnar nerve.

The likely diagnosis would not cause a motor deficit

The median nerve supplies muscles within the thenar eminence including the abductor pollicis brevis, flexor pollicis brevis and opponens pollicis and so there would be a motor deficit.

494 Rate this question: 1 A 65-year-old woman has presented to your clinic at almost 4 years following her right- sided mastectomy and axillary clearance. She had been transferred to your care from another hospital due to a change in address. On routine examination you note that she a winged right scapula. Which nerve roots are responsible for the likely pathology? 1. C7, 8 2. C5, 6 3. C6, 7, 8 and T1 4. C7, 8 and T1 5. C5, 6, 7 Explanation C5, 6, 7 An injury to the long thoracic nerve (C5, 6, 7), which can occur during weight lifting, thoracic surgery, or during an axillary lymph node clearance during a mastectomy, can result in paralysis of the serratus anterior muscle and ‘winging’ of the scapula. The nerve usually passes along the chest wall in the fascia over the serratus anterior, ie in the medial wall of the axilla. Patients usually experience minimal symptoms of posterior shoulder or scapular pain, which generally needs no further treatment. C7, 8 The long thoracic nerve is supplied by nerve roots C5, 6 and 7. C5, 6 The long thoracic nerve is supplied by C7 as well as C5 and 6. C6, 7, 8 and T1 The long thoracic nerve is not supplied by C8 or T1. C7, 8 and T1 The long thoracic nerve is supplied by C5, 6, and 7, not C8 or T1.

544 Rate this question: 1 2 3 4 5

You have been asked to see a young man in the Emergency Department who has been admitted following a fall out of a tree. To break his fall, he catches hold of a branch on the way down, jarring his shoulder. On presentation he shows signs of Klumpke’s paralysis. How would you describe this man’s neurological presentation?

1. It may be-associated with Horner’s syndrome and pupillary dilation on the ipsilateral side 2. It is often associated with eyelid retraction 3. The motor defect includes paralysis of the wrist extensors 4. The classic presentation is one of a claw hand 5. The sensory deficit is in the distribution of the median nerve Explanation The classic presentation is one of a claw hand Klumpke’s syndrome is a partial palsy of the lower roots of the brachial plexus (C8 and T1) after or before they have joined to form the lower trunk. This is a rare condition and has a variable recovery time and some may never resolve (especially following birth injuries). The paralysis affects the intrinsic muscles of the hand (notably the interossei, thenar and hypothenar muscles) and the flexors of the wrist and fingers (notably flexor carpi ulnaris and flexor digitorum). Forearm pronators and wrist flexors may also be affected. The clinical picture is the claw hand where the forearm is supinated and the wrist and fingers are flexed. The sensory deficit is in the ulnar nerve distribution and a Horner’s syndrome with ptosis and miosis (pupillary constriction) may also be seen due to T1 involvement.

It may be-associated with Horner’s syndrome and pupillary dilation on the ipsilateral side

If there is T1 involvement, Horner’s syndrome may be present with pupillary constriction (miosis), not dilation. It is often associated with eyelid retraction Eyelid retraction is not seen, instead ptosis is often present. The motor defect includes paralysis of the wrist extensors Wrist flexors, not extensors may be affected with Klumpke’s syndrome. The sensory deficit is in the distribution of the median nerve Sensory deficit is seen in the distribution of the ulnar, not median nerve.

549 Rate this question: 1 2 3 4 5 A 32-week pregnant woman presents to your clinic with disabling tingling in the thumb, index, middle and ring fingers of her dominant hand. She is dropping light items and cannot grip a cup of tea. Which one of the following statements best represents her likely pathology?

1. Symptoms are a recognised complication of osteoarthritis 2. Symptoms are frequently caused by a cervical rib 3. The abductor pollicis brevis will be spared 4. The diagnosis can be made with confidence on review of nerve conduction studies 5. Treatment is always by division of the extensor retinaculum

Explanation The diagnosis can be made with confidence on review of nerve conduction studies The likely diagnosis is carpal tunnel syndrome. The causes of carpal tunnel include pregnancy, rheumatoid arthritis, hypothyroidism, and fractures in the region of the wrist. The syndrome results from compression of the median nerve in the tunnel under the flexor retinaculum and if conservative measures fail, treatment is by surgical division of the retinaculum. Nerve conduction studies will demonstrate a reduced conduction velocity across flexor retinaculum in the median nerve indicating compression given the clinical history. Conduction studies of the radial and ulnar nerve will be normal. Importantly, while nerve conduction can confirm the diagnosis often in primary care the diagnosis is clinical.

Symptoms are a recognised complication of osteoarthritis

Rheumatoid arthritis is more commonly associated with carpal tunnel syndrome due to the associated synovial inflammation seen with rheumatoid disease.

Symptoms are frequently caused by a cervical rib

Cervical rib, as well as cervical spondylosis, could be included in the differential diagnosis for this presentation, however, this patient only has symptoms distal to the wrist, these other differentials would produce symptoms and sign proximal to the wrist. The abductor pollicis brevis will be spared The median nerve supplies motor fibres to the abductor policies brevis muscle. Treatment is always by division of the extensor retinaculum Treatment options for carpal tunnel syndrome include conservative measures, such as physiotherapy, corticosteroid injections and subsequently surgery if conservative measures fail. Surgery involves division of the flexor, not extensor, retinaculum. 557 Rate this question: 1 2 A 27-year-old woman is thrown from a horse she is riding and lands heavily on her left shoulder. She has no other injuries but reports difficulty in moving her left shoulder and elbow. On examination she is unable to abduct the left shoulder or flex the elbow, and the shoulder is medially rotated with the palm facing posterior. Which one of the following best describe this clinical pattern? 1. Claw hand 2. Erb’s palsy 3. Klumpke’s palsy 4. Horner’s syndrome 5. Sudeck’s atrophy Explanation Erb’s palsy Most brachial plexus injuries are due to severe traction, such as the trauma described in this scenario, however Erb’s palsy is most commonly in neonates following traumatic birth injuries. Of note, fractures of the clavicle rarely damage the plexus. An Erb’s palsy is caused by damage to C5, C6 and sometimes C7 roots. In an injury to the superior trunk of the brachial plexus the characteristic portion of the limb is a ‘waiter’s tip position’ in which there is weakness in shoulder abduction, external rotation, elbow flexion and supination. This leads to the limb being held in the described position. Claw hand Claw hand is more commonly seen with Klumpke’s palsy. Klumpke’s palsy A Klumpke’s palsy is due to damage to C8 andT1, and gives rise to weak wrist and finger flexors, intrinsic hand muscles are paralysed with loss of sensation in the ulnar forearm and hand. There may be an associated Horner’s syndrome. This palsy is most commonly encountered in neonates following traumatic birth injuries. Horner’s syndrome Horner’s syndrome occurs due to a deficiency of sympathetic activity and develops symptoms on the ipsilateral side to the lesion. The characteristic features are ptosis miosis, anhidrosis and enophthalmos. Sudeck’s atrophy Sudeck’s atrophy is another name for type 1 complex regional pain syndrome.

568 Rate this question: 1 2 3 4 5

A 27-year-old rugby player has undergone surgery for a comminuted proximal humerusfracture in which it is noted anatomical reduction was very difficult. Six months following surgery it is noted he has weakness of elbow flexion and altered sensation over the lateral aspects of his forearm. No other areas of weakness or sensory deficit are noted. Which one of the following nerves is likely to be injured? 1. Axillary 2. Median 3. Musculocutaneous 4. Radial 5. Ulnar Explanation Musculocutaneous The musculocutaneous nerve supplies the coracobrachialis, biceps brachii and brachialis muscles. It pierces the deep fascia just proximal to the elbow joint and becomes superficial. It is then called the lateral cutaneous nerve of the forearm, supplying skin on the lateral aspect of the forearm. It is probable that, due to the difficult reduction of the fracture intraoperatively, the musculocutaneous nerve has been damaged. Axillary The axillary nerve can be damaged following anterior–inferior dislocations of the shoulder and fractures of the surgical neck of humerus, however this would present with weakness in shoulder abduction and weak flexion, extension, and rotation of the shoulder. Median By far the most common cause of median nerve palsy is carpal tunnel syndrome, however supracondylar elbow fractures can also cause a median nerve palsy. It is rarely injured at the level of the proximal humerus. Radial The radial nerve innervates the posterior compartment of the upper arm and forearm. A radial nerve palsy would present with weakness in forearm extension, loss of extension of the fingers and wrist drop. These features are not described above, moreover the radial nerve is more commonly injured following a mid-shaft humeral fracture. Ulnar The ulnar nerve is more commonly injured at the elbow and would present with weakness in flexion of the hand at the wrist and a claw hand deformity at rest.

601 Rate this question: 1 2 3 4 5 A 24-year-old soldier sustained a gunshot wound to the right proximal thigh. Neurological examination revealed he had an absent knee jerk reflex, numbness over the medial and anterior thigh, and medial malleolus. The cremasteric reflex is intact. Which is the most likely nerve to have been injured? 1. Femoral nerve 2. Genitofemoral nerve 3. Lateral cutaneous nerve of the thigh 4. L2 nerve root 5. Saphenous nerve Explanation Femoral nerve

Femoral nerve injury is rare and usually secondary to stab or gunshot wounds. Motor fibres supply the quadriceps femoris muscle, the contracting fibres of which elicit the knee jerk. The nerve supplies the L2–L4 dermatomes over the anterior skin of the thigh. The genitofemoral nerve mediates the cremasteric reflex. Genitofemoral nerve

The genitofemoral nerve supplies the L2 dermatome and damage to this nerve would not impact on the knee jerk reflex. Lateral cutaneous nerve of the thigh

The lateral cutaneous nerve of the thigh supplies the L2 dermatome, but again damage to this nerve would not impact on the knee jerk reflex. L2 nerve root

Given the anatomical location of the gunshot wound, damage to the L2 nerve root is unlikely. Saphenous nerve The saphenous nerve (L4), a branch of the femoral nerve, innervates the skin over the medial malleolus, however an isolated saphenous nerve injury would not affect the knee jerk reflex as it is distal to the femoral nerve.

679 Rate this question: 1 2 3 4 5

A 28-year-old man with a long standing history of back pain presents acutely with shooting pains to the outer aspect of his right foot, reduced sensation to the sole of the right foot, weakness of plantar flexion of the right ankle and an absent ankle jerk reflex on the right side. There is no bladder or bowel incontinence. Which nerve root is most likely to be compressed?

1. Cauda equina 2. Left S1 nerve root 3. Right L1 nerve root 4. Right L5 nerve root 5. Right S1 nerve root Explanation Right S1 nerve root Signs of S1 nerve root compression include reduced sensation in the S1 dermatome (sole of the foot), weakness of plantar flexion of the ankle, and absent or reduced ankle jerk.

Cauda equina

As this is a unilateral problem and there is no evidence of bowel or bladder incontinence then there is no evidence of cauda equina. Left S1 nerve root

The symptoms and signs described are present on the right, not the left. Decussation takes place in the spinal cord, therefore signs and symptoms are present are ipsilateral to the affected nerve root. Right L1 nerve root

Compression of the L1 nerve root would produce pain in the groin. Right L5 nerve root

Compression of the L5 nerve root may present with pain in a similar distribution to that of S1 compression, however, in L5 compression the great toe and medial border of the foot is affected, not the lateral aspect.

680 Rate this question: 1 2 3 4 5

You are in the neurosurgical clinic reviewing a patient who has had a left temporal lobe tumour resected and now has a suspected abscess. He has raised inflammatory markers, fever and headache. A dominant hemisphere temporal lobe abscess is most likely to cause which symptom? 1. Motor disturbance in the contralateral arm 2. Problems with balance 3. Problems with speech (dysphasia) 4. Sensory disturbance in the contralateral leg 5. Inferior homonymous quadrantanopia Explanation Problems with speech (dysphasia) Cerebral abscesses act as space-occupying lesions and produce focal neurological deficits. They may also cause meningitis, epilepsy and intracranial herniation. Dysphasia occurs with dominant (most commonly left) temporal lobe lesions. The ‘important’ cortical areas of the temporal lobe area are the auditory cortex and temporal association area (responsible for the recognition of auditory stimuli and integration with other modalities). Motor disturbance in the contralateral arm The primary motor cortex is located at the posterior border of the frontal lobe, given this patient’s abscess is located in the temporal lobe, motor function is likely to be preserved. Problems with balance Issues with balance are more likely to occur with disorders of the cerebellum. Sensory disturbance in the contralateral leg The sensory cortex is located in the parietal lobe and is unlikely to be affected by temporal lobe pathology. Inferior homonymous quadrantanopia A temporal lobe lesion would produce an upper homonymous quadrantanopia, while a parietal lobe lesion would cause an inferior homonymous quadrantanopia.

706 Rate this question: 1 2

3 4 5

Following a left carotid endarterectomy a 74-year-old patient is noted to have developed speech problems. He seems able to understand what is said but unable to get the words out. Which area of the brain is most likely to be affected? 1. Basal ganglia 2. Broca’s area 3. Occipital cortex 4. Olfactory cortex 5. Wernicke’s area Explanation Broca’s area

The primary motor cortex is in the precentral gyrus and the primary sensory cortex in the postcentral cortex. Broca’s area lies in the posterior part of the inferior frontal gyrus of the dominant hemisphere. Lesions to Broca’s area cause expressive dysphasia, classically described as knowing what to say, but being unable to get the words out. Basal ganglia

The putamen and globus pallidus are together known as the lentiform nucleus and form part of the basal ganglia. Damage to the basal ganglia produces movement disorders. Occipital cortex

The occipital cortex functions as the primary visual cortex, and there are no visual disturbances described in the case history. Olfactory cortex

The olfactory impulses travel to the temporal lobe in the region of the uncus, and no olfactory disturbances are described. Wernicke’s area The Wernicke’s area is tasked with the comprehension of speech, and damage to this area would result in a receptive dysphasia, however, this man has an expressive dysphasia suggesting a damage to Broca’s area.

719 Rate this question: 1 2 3 4

5

Different areas of the brain are responsible for different functions. The presenting neurological symptoms and signs a patient may have can point towards where the lesion is. Lesions in the occipital area of the brain may cause which one of the following? 1. Personality changes 2. Hemiparesis 3. Non-fluent aphasia 4. Visual field defects 5. Urinary incontinence Explanation Visual field defects

Visual field defects are usually associated with lesions in the occipital region of the brain. Personality changes Lesions of the frontal lobe are associated with personality changes, urinary incontinence and mono or hemiparesis. Hemiparesis

Hemiparesis occurs as a result of damage to the primary motor cortex which is located in the frontal lobe. Non-fluent aphasia

Non-fluent aphasia occurs in lesions of the left frontal lobe. Urinary incontinence

Damage to the frontal lobe can result in urinary incontinence, probably the best example of this is frontotemporal dementia which often presents with significant cognitive impairment and incontinence.

785 Rate this question: 1 2 3 4 5

Different areas of the brain are responsible for different functions. The presenting neurological symptoms and signs a patient may have can point towards where the lesion is. Lesions in the occipital area of the brain may cause which one of the following? 1) Personality changes 2) Hemiparesis 3) Non-fluent aphasia 4) Visual field defects 5) Urinary incontinence Explanation Visual field defects

Visual field defects are usually associated with lesions in the occipital region of the brain. Personality changes Lesions of the frontal lobe are associated with personality changes, urinary incontinence and mono or hemiparesis. Hemiparesis

Hemiparesis occurs as a result of damage to the primary motor cortex which is located in the frontal lobe. Non-fluent aphasia

Non-fluent aphasia occurs in lesions of the left frontal lobe. Urinary incontinence

Damage to the frontal lobe can result in urinary incontinence, probably the best example of this is frontotemporal dementia which often presents with significant cognitive impairment and incontinence.

785 Rate this question:

1 2 3 4 5

You are asked to see a patient with new onset neurological deficits. By the time you go down to assess them they seem to be markedly improving compared to the documentation by the A&E SHO. Which one of the following is the commonest cause of a transient ischaemic attack?

1. Thrombophilia 2. Atrial fibrillation 3. Myocardial infarction 4. Paradoxical embolus 5. Atherosclerosis of the carotid bifurcation Explanation

Atherosclerosis of the carotid bifurcation

Although all the above may give rise to embolisation within the arterial tree travelling to the cerebral circulation and presenting as a transient ischaemic attack, a 70% stenosis of the carotid bifurcation or proximal internal carotid artery is by far the commonest cause. Ultrasound imaging of the carotid arteries following a TIA is important, and carotid endarterectomy may be indicated in some patients. Secondary prevention is also important with medications to control blood pressure and cholesterol.

Thrombophilia

Thrombophilia is less common than atherosclerosis of the carotid arteries as a cause of transient ischaemic attacks (TIA).

Atrial fibrillation

Anticoagulation is utilised to reduce the risk of TIA and stroke in atrial fibrillation due to the increased risk of thromboembolism, however atherosclerosis remains a more common cause for TIA.

Myocardial infarction

Myocardial infarction may result in the production of mural thrombus, which may embolise through the arterial tree and occlude the cerebral circulation, however, atherosclerosis of the carotid artery is a more common cause.

Paradoxical embolus

Atherosclerosis of the carotid artery is a more common cause of a transient ischaemic attack (TIA). 966 Rate this question: A patient is admitted with new onset neurological signs. CT head demonstrates a posterior circulation stroke. Cerebellar lesions produce which of the following?

1. Waddling gait 2. Festinant gait 3. Ataxic gait 4. Scissors gait 5. High-stepping gait Explanation

Ataxic gait

In disease of the lateral cerebellar lobes, the stance becomes broad based, unstable and tremulous ie ataxic. The gait tends to veer towards the side of the more affected cerebellar lobe. The underlying causes of cerebellar ataxia are extensive but can include; multiple sclerosis, infection, infarction, vasculitis and autoimmune conditions.

Waddling gait

Weakness of proximal lower limb muscles (eg in polymyositis or muscular dystrophy) leads to difficulty in rising from sitting or squatting. Once upright, the patient walks with a waddling gait, as each lower limb, as it carries the full weight of the body, does not adequately support the pelvis.

Festinant gait

Festinant, or hurried gait occurs in Parkinson’s disease.

Scissors gait

Spasticity causes stiffness and jerkiness while walking – scissors gait.

High-stepping gait

Broad-based, high stepping or stamping gait develops in peripheral sensory lesions (eg polyneuropathy) when there is loss of proprioception.

1004 Rate this question: 1 2 3 4 5

A 55-year-old man, with a two year history of back pain presents with worsening symptoms. He has been referred to the acute spinal service for urgent assessment. Which one of the following signs is most suggestive of acute cauda equina syndrome? 1. Muscle wasting 2. Hyperactive lower limb reflexes 3. Loss of sensation to the lateral aspect of the foot 4. Loss of perianal tone 5. Positive Babinski sign Explanation Loss of perianal tone CES is caused when there is compression of the lumbosacral nerves below the level of the spinal cord. It typically presents with low back pain, unilateral or bilateral sciatica, bladder or bowel dysfunction and perianal sensory loss or hypoaesthesia. Loss of perianal tone is characteristic of CES. Muscle wasting Muscle wasting suggests chronic cauda equina syndrome (CES), this question is specifically discussing acute CES. Hyperactive lower limb reflexes Hyperactive lower limb reflexes and a positive Babinski sign are upper motor neurone signs and therefore make the diagnosis of CES unlikely. Loss of sensation to the lateral aspect of the foot Although sensory changes may be noted in acute cauda equina syndrome this affects the saddle area (S3–S5). Loss of sensation to the lateral foot would implicate S1–S2. Positive Babinski sign A positive Babinski is an upper motor neurone sign and therefore may the diagnosis of CES unlikely.

1005 Rate this question: 1 2 3 4 5

A 53-year-old man who suffers from long standing alcoholism has had a history of multiple falls over the last 2 months. He complains of a progressive headache. A computed tomography (CT) scan does not reveal any skull fractures. Which other findings is this patient most likely to have? 1. Air in the cranial cavity 2. Anosmia 3. Gait disturbance 4. Meningitis 5. Rhinorrhoea

Explanation Gait disturbance Chronic subdural haematomas are most commonly found in the older age groups. Chronic alcohol ingestion is also an associated factor due to the frequent incidence of falls. Chronic subdural haematomas are collections of altered blood and are most frequently triggered by minor trauma. The symptoms include progressive headache, gait disturbance, failing intellect, hemiparesis and a fluctuating conscious level. Air in the cranial cavity Air in the cranial cavity would suggest a significant skull fracture and the CT scan has ruled this out. Anosmia Anosmia is associated with acute subdural haematoma and traumatic brain injury. Meningitis Meningitis is an acute presentation with worsening headache, neck stiffness and photophobia. If not treated early and aggressively is associated with high mortality and morbidity. Rhinorrhoea Acute subdural haematoma is more commonly associated with anosmia and rhinorrhoea as there is usually a concomitant skull fracture. 1008 Rate this question: 1 2 3 4 5

A 33-year-old man involved in an altercation is brought to the Emergency Department and is diagnosed with an extradural haematoma. His GCS has remained stable at 14 since admission and his case has been referred urgently to the local neurosurgical centre. Which anatomical structures are most likely to limit the spread of the haematoma?

1. Intracranial venous sinuses 2. The falciform fold 3. Suture lines 4. Tentorium cerebelli 5. The ventricular space

Explanation

Suture lines

Extradural haematoma is associated with fracture and is usually the result of damage to the anterior branch of the middle meningeal artery. The dura has strong attachments to the cranium along the suture lines and as a result, these attachments can limit the extent of the haematoma.

Intracranial venous sinuses

Due to the arterial nature of an extradural haematoma, the venous sinuses with their much lower blood pressure will not limit an expanding haematoma.

The falciform fold

The falciform fold is a congenital fold from the disc to the ciliary region of the retina and is unlikely to play any role in limiting the expansion of an extradural haematoma.

Tentorium cerebelli

The tentorium cerebelli is an extension of the dura mata separating the inferior parietal lobes from the cerebellum. An expanding haematoma supratentorially is not going to be limited by the tentorium cerebelli, indeed, by the time the tentorium cerebelli limits the expansion uncal herniation and compression of the brainstem will have already occurred. The ventricular space The ventricular space in an extradural haematoma will compress allowing expansion, not limitation, of the haematoma. 1009 Rate this question: 1 A 16-year-old girl is admitted to the Emergency Department having fallen and injured her head while out with friends. She is clearly intoxicated. Her friends say she passed out, and the patient does not recall what happened. Which one of the following statements reflects NICE guidelines regarding the management of head injuries?

1. All patients who have had loss of consciousness (LOC) must be admitted 2. All patients who have had LOC need a CT scan 3. All intoxicated patients with LOC must be admitted 4. All patients with a Glasgow Coma Scale (GCS) < 13 at any point since the injury should have a CT scan 5. All patients with continuing symptoms such as nausea or vomiting, and brief lapses in memory can go home provided they are accompanied and can be observed by a responsible adult Explanation

All patients with a Glasgow Coma Scale (GCS) < 13 at any point since the injury should have a CT scan

The following reflects the NICE guidelines for head injuries and their management (CG176 January 2014). Patients who have sustained a head injury and present with any one of the following risk factors should have computed tomography (CT) scanning of the head immediately requested:

 Glasgow Coma Scale (GCS) less than 13 on initial assessment in the Emergency Department  GCS less than 15 at 2 h after the injury  suspected open or depressed skull fracture  any sign of basal skull fracture (haemotympanum, ‘panda’ eyes, cerebrospinal fluid, otorrhoea, Battle’s sign)  post-traumatic seizure  focal neurological deficit  more than one episode of vomiting  for head injuries in children who have none of the above but more than one of the following:  loss of consciousness lasting more than 5 mins (witnessed)  abnormal drowsiness  dangerous mechanism of injury eg traffic accident at high speed  amnesia (antegrade or retrograde lasting more than 5 mins (this is not possible in preverbal children and very difficult in any child under 5). All patients who have had loss of consciousness (LOC) must be admitted

The following patients meet the criteria for admission to hospital following a head injury:

 Patients with new, clinically significant abnormalities on imaging.  Patients who have not returned to GCS 15 after imaging, regardless of the imaging results.  When a patient fulfils the criteria for computed tomography (CT) scanning, but this cannot be done within the appropriate period, either because CT is not available or because the patient is not sufficiently co-operative to allow scanning.  Continuing worrying signs of concern to the clinician (for example, persistent vomiting, severe headaches).  Other sources of concern to the clinician (eg drug or alcohol intoxication, other injuries, shock, suspected non-accidental injury, meningism, cerebrospinal fluid leak).

As such, not all patients who have experienced a loss of consciousness will require admission.

All patients who have had LOC need a CT scan

A single episode of loss of consciousness is common following a head injury and does not warrant CT scanning according to NICE guidelines.

All intoxicated patients with LOC must be admitted

An intoxicated patient may warrant admission for observation following a head injury only if there is significant concern from the assessing clinician, however, this may mean some patients will not require admission.

All patients with continuing symptoms such as nausea or vomiting, and brief lapses in memory can go home provided they are accompanied and can be observed by a responsible adult

The following patients meet the criteria for admission to hospital following a head injury:

 Patients with new, clinically significant abnormalities on imaging.  Patients who have not returned to GCS 15 after imaging, regardless of the imaging results.  When a patient fulfils the criteria for CT scanning but this cannot be done within the appropriate period, either because CT is not available or because the patient is not sufficiently co-operative to allow scanning.  Continuing worrying signs of concern to the clinician (for example, persistent vomiting, severe headaches).  Other sources of concern to the clinician (eg drug or alcohol intoxication, other injuries, shock, suspected non-accidental injury, meningism, cerebrospinal fluid leak).

1011 A paraganglioma is a rare neuroendocrine neoplasm which presents as a painless mass. You are discussing this diagnosis with a patient in clinic. Which one of the following options is a correct description of a paraganglioma? 1. They are secretory 2. They arise from non-chromaffin paraganglionic cells and commonly metastasise 3. They arise from preganglionic chromaffin cells derived from the neural crest 4. They may be associated with pheochromocytomas 5. They only occur in the parasympathetic nervous system Explanation

They may be associated with pheochromocytomas

Paragangliomas are carotid body tumours which arise from nests of non-chromaffin paraganglionic cells derived from the neural crest. There is a high incidence in the people of Peru (altitude 2000–5000 m), Mexico City and Colorado. In addition to bilateral tumours and associated phaeochromocytomas there is a strong family tendency. They are secretory Paragangliomas are non-secretory. They arise from non-chromaffin paraganglionic cells and commonly metastasise They arise from non-chromaffin paraganglionic cells and rarely metastasise. They arise from preganglionic chromaffin cells derived from the neural crest They arise from non-chromaffin paraganglionic cells, not preganglionic chromaffin cells. They only occur in the parasympathetic nervous system They can be seen in both the sympathetic and parasympathetic nervous system.

1015 Rate this question: 1 2 3 4 5 A patient is referred to the general surgical clinic with excessive sweating. She is considered for operative intervention. Regarding hyperhidrosis, which one of the following options is true?

1. If affecting the axilla it may be treated with thoracoscopic ablation of the T1 and T2 ganglia 2. If affecting the palms it may be treated with thoracoscopic ablation of the T4 and T5 ganglia 3. It may be treated with intra-dermal Botox injections 4. It may be treated with oral 20% aluminium chloride hexahydrate 5. It most commonly affects the chest Explanation

It may be treated with intra-dermal Botox injections

Patients most often complain of excessive sweating of the palms, soles and axillae. Topical rather than oral aluminium chloride may be used as well as intra-dermal Botox injections.

If affecting the axilla it may be treated with thoracoscopic ablation of the T1 and T2 ganglia

The first (T1) thoracic ganglion controls facial sweating, the second (T2) and third (T3) thoracic ganglia are responsible for palmar sweating.

If affecting the palms it may be treated with thoracoscopic ablation of the T4 and T5 ganglia

The fourth (T4) thoracic ganglion controls axillary sweating.

It may be treated with oral 20% aluminium chloride hexahydrate

Topical rather than oral aluminium should be used to reduce systemic side-effects. It most commonly affects the chest The most commonly affected areas are the palms, soles of the feet and axillae, not the chest.

1668 Rate this question: 1 2 3 4 5

You are consenting a patient for thoracoscopic sympathectomy. They have high expectations for a successful procedure. Which one of these complications is the most common reason for dissatisfaction following this operation? 1. Bradycardia 2. Compensatory sweating (hyperhidrosis affecting previously uninvolved areas of the body) 3. Horner syndrome 4. Intercostal neuralgia 5. Pneumothorax Explanation

Compensatory sweating (hyperhidrosis affecting previously uninvolved areas of the body)

The cause of compensatory and gustatory sweating remains an enigma and it is important to make this possibility clear to the patient before operation. It should be noted that this is a controversial procedure and outcomes are not predictable and extremely variable. Bradycardia Thoracoscopic sympathectomy can cause bradycardia but this not commonly symptomatic.

Horner syndrome

Horner syndrome results if the stellate ganglion is accidentally injured, however, this is uncommon.

Intercostal neuralgia

Intercostal neuralgia is not a common post operatively. Pneumothorax As the pleural cavity is entered during this procedure a small pneumothorax does occur, however, this will usually resolve without intervention following the operation and is not a common cause for dissatisfaction.

1679 Rate this question: 1 2 3 4 5

You are treating a patient with a pituitary tumour, which is affecting the posterior portion of the gland on CT scans. They are beginning to develop systemic manifestations of the disease. Secretion of which hormone is most likely to be affected by this tumour? 1. Adrenocorticotropic hormone (ACTH) 2. Follicle stimulating hormone (FSH) 3. Oxytocin 4. Prolactin 5. Thyroid stimulating hormone (TSH) Explanation Oxytocin The posterior pituitary secretes oxytocin and antidiuretic hormone (ADH) into the bloodstream. Oxytocin stimulates lactation in response to suckling. Adrenocorticotropic hormone (ACTH) The anterior pituitary secretes adrenocorticotropic hormone (ACTH), growth hormone, thyroid-stimulating hormone (TSH), follicle-stimulating hormone (FSH), prolactin and LH. The secretion of these is governed by releasing and inhibitory factors released into the pituitary portal system. The hypothalamus regulates levels of these specific factors by monitoring circulating levels of hormones produced by the target organ. For example – levels of FSH and luteinizing hormone (LH) are controlled by luteinizing hormone- releasing hormone (LHRH) from the hypothalamus in response to circulating levels of oestrogens and androgens.

Follicle stimulating hormone (FSH)

As discussed, FSH is secreted by the anterior pituitary. Prolactin Prolactin is secreted by the anterior pituitary. Thyroid stimulating hormone (TSH) TSH is secreted by the anterior pituitary.

1694 Rate this question: 1 2 3 4 5

A 50-year-old man with a long-standing history of alcohol abuse was admitted to hospital because he was difficult to rouse. On examination he is confused and ataxic. Examination of the eyes reveals nystagmus and ophthalmoplegia. Deficiency of which vitamin is most likely to have caused his symptoms? 1. Vitamin A

2. Vitamin B1

3. Vitamin B6

4. Vitamin B12 5. Folic acid Explanation

Vitamin B1 A common cause of the amnesic syndrome is the Wernicke–Korsakoff syndrome resulting from thiamine (vitamin B1) deficiency in association with chronic alcoholism or, occasionally, malnutrition or malabsorption. The Wernicke phase of this disorder is characterised by confusion, nystagmus, abducent and conjugate gaze palsies (ophthalmoplegia) as well as ataxia. These features are commonly accompanied by . Prompt treatment with thiamine replacement is vital to avert a chronic and disabling amnesic disorder (the Korsakoff syndrome). Vitamin A Vitamin A deficiency is a common cause of night blindness in the developing world but is rare in the developed world. The case history is not consistent with vitamin A deficiency.

Vitamin B6 Again, while chronic alcoholism is a risk for micro-nutrient and vitamin deficiencies the case history described is that of Wernicke–Korsakoff syndrome.

Vitamin B12

Vitamin B12 deficiency most commonly results in macrocytic anaemia, rarely, neurological complications can occur. Interestingly, chronic alcoholism can cause a mild macrocytosis even in the absence of B12 deficiency. Folic acid Folate deficiency is accelerated by alcoholism and clinical presentation can be varied, however, it would not be associated with nystagmus or ophthalmoplegia.

2138 Rate this question: 1 2 3 4 5

You are in the Emergency Department reviewing an elderly male patient with sudden onset left arm weakness and upper motor neuron signs which resolved over a few minutes. He has a bruit over his left carotid on careful examination. Which one of the following regarding transient ischaemic attack (TIA) is correct? 1. Increases the risk of future stroke 2. An indication for carotid endarterectomy if associated with a less than 70% stenosis of the appropriate internal carotid artery 3. Any neurological deficit lasting more than 2 h 4. Is best treated with antiplatelet agents only 5. Most commonly results from cerebral hypo-perfusion secondary to a very tight stenosis in the carotid artery

Explanation Increases the risk of future stroke The risk is of further stroke is increased and brain imaging, ideally a magnetic resonance imaging (MRI) scan and an urgent carotid Doppler test are needed.

An indication for carotid endarterectomy if associated with a less than 70% stenosis of the appropriate internal carotid artery

Carotid endarterectomy is indicated if there is a stenosis of greater than 70%.

Any neurological deficit lasting more than 2 h

A TIA is a focal neurological deficit lasting less than 24 h. Is best treated with antiplatelet agents only A combination of antiplatelet therapy and endarterectomy may be indicated depending on the severity of the stenosis and whether it corresponds to the clinical syndrome. Most commonly results from cerebral hypo-perfusion secondary to a very tight stenosis in the carotid artery When in the area of the carotid artery it is most commonly due to embolisation from atheromatous plaque at the carotid bifurcation. 2184 Rate this question: 1 2 3 4 5

A middle-aged patient is admitted to A&E resus and the acute stroke team are asked to assess them urgently. They have developed dysarthria, nystagmus and a tremor that worsens with directed movement. This patient most probably has which of the following?

1. Cerebellar disease 2. Damage to pontine and caudate nuclei 3. Hyperthyroidism 4. Parkinsonism 5. Spinal cord transection Explanation

Cerebellar disease

The patient in this description has cerebellar tremor as suggested by the signs and symptoms. Cerebellar tremor is a slow, broad tremor of the extremities that occurs at the end of a purposeful movement, such as trying to press a button or touching a finger to the tip of one’s nose. Cerebellar tremor is caused by lesions in or damage to the cerebellum resulting from stroke, tumour or disease such as multiple sclerosis or some inherited degenerative disorder. It can also result from chronic alcoholism or overuse of some medicines. In classic cerebellar tremor, a lesion on one side of the brain produces a tremor in that same side of the body that worsens with directed movement. Cerebellar damage can also produce a ‘wing-beating’ type of tremor called rubral or Holmes’ tremor – a combination of rest, action and postural tremors. The tremor is often most prominent when the affected person is active or is maintaining a particular posture. Cerebellar tremor may be accompanied by dysarthria (speech problems), nystagmus (rapid, involuntary rolling of the eyes), gait problems and postural tremor of the trunk and neck.

Damage to pontine and caudate nuclei

Damage to the pontine and caudate nuclei would cause a resting tremor that lessens on voluntary movement, which is not the case described. It is also unlikely to be associated with nystagmus and dysarthria.

Hyperthyroidism

Tremor may be present in hyperthyroidism, however, it would not be associated with dysarthria or nystagmus.

Parkinsonism

The tremor associated with parkinsonism is a resting tremor that ceases on voluntary movement, this is contrary to the tremor described in the case history that worsens on voluntary movement.

Spinal cord transection Spinal cord transection is more commonly caused by significant trauma and is associated with focal neurological deficit (such as paralysis and paraesthesia) below the level of the injury.

2193 Rate this question: 1 2 3 4 5

You are seeing a patient in the neurosurgical clinic with a posterior cerebral artery infarction. They have debilitating symptoms and are unable to lead a normal life. What symptoms would you expect to find? 1. Nystagmus 2. Dysdiadochokinesis 3. Dysphasia 4. Loss of light reflex 5. Homonymous hemianopia Explanation Homonymous hemianopia The posterior cerebral artery supplies the cerebral peduncle and the optic tract. It supplies the inferomedial surface of the temporal and occipital lobes. Homonymous hemianopia is caused by a lesion beyond the optic chiasm opposite to the field defect, eg due to stroke or a tumour. Nystagmus Nystagmus and dysdiadochokinesis are cerebellar signs. Dysdiadochokinesis Nystagmus and dysdiadochokinesis are cerebellar signs.

Dysphasia

Speech would be unaffected as Broca’s and Wernicke’s areas, which are responsible for the comprehension and production of speech, are located in the frontal and temporal lobes which are largely supplied by the middle cerebral artery.

Loss of light reflex

The light reflex is a primitive reflex which circuits through the midbrain and therefore would be unaffected by a posterior cerebral artery territory infarction.

2204 Rate this question: 1 2 3 4 5

You are assessing a patient who has suffered a blunt head injury approximately 2 h ago. You suspect raised intracranial pressure. Which one of the following signs would be most consistent with this diagnosis? 1. Bloody tap following lumbar puncture 2. Decreased blood pressure 3. Increased pulse rate 4. Irregular breathing 5. Neck stiffness Explanation Irregular breathing Rising intracranial pressure presents with headache, drowsiness, vomiting and seizures and there is often a history of trauma. Signs include listlessness, irritability, drowsiness, falling pulse, rising blood pressure, coma, irregular breathing (respiratory depression from compression of the medulla) and, later, papilloedema. Cushing’s triad also indicates raised intracranial pressure; bradycardia, hypertension and irregular respiration. Bloody tap following lumbar puncture Lumbar puncture is contraindicated in raised intracranial pressure. Decreased blood pressure Hypertension, not hypotension, is associated with increased intracranial pressure as part of Cushing’s triad; hypertension, bradycardia and irregular respiration. Increased pulse rate As discussed, raised intracranial pressure can result in Cushing’s triad; bradycardia, hypertension and irregular respiration. Neck stiffness Neck stiffness is associated more commonly with meningitis, not raised intracranial pressure.

2217 Rate this question: 1 2 3 4 5

A 21-year-old man comes off his motorbike at 65 mph and is admitted with a large subdural haematoma. He undergoes intracranial pressure monitoring. The early phase of increased intracranial pressure (ICP) causes which one of the following? 1. Decreased pulse pressure 2. Hyperventilation 3. Hypotension 4. Pupillary constriction 5. Tachycardia

Explanation Pupillary constriction Pupils constrict initially due to external compression of overlying sympathetic fibres, but later dilate when the oculomotor nerve is compressed. Complete third nerve palsy may also occur. As the herniation progresses, the contralateral oculomotor nerve may be compressed, producing bilateral pupil dilation. Decreased pulse pressure Raised intracranial pressure may give rise to Cushing’s triad with decrease in respiratory rate, bradycardia, hypertension and increased pulse pressure. Hyperventilation Rising intracranial pressure may result in Cheyne–Stokes respiration and subsequent respiratory depression, but not hyperventilation. Hypotension As discussed, hypertension (as part of Cushing’s triad) may ensue, not hypotension. Tachycardia Bradycardia is associated with rising intracranial pressure. 2225 Rate this question: 1 2 3 4 5

A 37-year-old man is admitted with headache and papilloedema. An urgent computed tomography (CT) scan is requested. Which one of the following is true of a space-occupying lesion in the left cerebral hemisphere? 1. May reduce intracranial pressure 2. May shift the third ventricle to the left 3. May cause nausea and vomiting 4. Must be large before it causes symptoms 5. Rarely presents with symptoms

Explanation May cause nausea and vomiting Nausea and vomiting are signs of raised ICP. May reduce intracranial pressure The skull is a rigid compartment and its three main contents (brain, blood and cerebrospinal fluid) are in dynamic equilibrium (Monroe–Kellie doctrine). An uncompensated increase in one component will result in raised intracranial pressure (ICP). May shift the third ventricle to the left Space-occupying lesions cause a shift of cerebral matter to the opposite side and so may compress the lateral ventricle. Must be large before it causes symptoms Depending on location, small lesions can cause symptoms. Rarely presents with symptoms Relatively large lesions may develop in the frontal lobes before any signs of raised ICP are seen, however, other symptoms such as nausea and headache are likely to be present. 2226 Rate this question: 1 2 3 4 5

A 34-year-old woman is referred to an endocrine clinic with amenorrhoea for the last 9 months after giving birth. It was initially thought to be secondary to breast feeding but this was stopped 4 months ago. Blood tests reveal several other abnormalities including hyponatraemia, low thyroid-stimulating hormone (TSH) and thyroid hormone levels. Her only past medical history of note was very heavy post-partum haemorrhage requiring a massive transfusion. What is the most likely diagnosis in this case? 1. Empty-sella syndrome 2. Kallmann’s syndrome 3. Pituitary apoplexy 4. Sheehan syndrome 5. Subacute thyroiditis Explanation Sheehan syndrome Sheehan’s syndrome is also known as Simmonds’ syndrome or post-partum pituitary gland necrosis. It is now a rare condition which occurs following pituitary infarction due to severe post-partum haemorrhage. The pituitary gland is vulnerable as it relies on a low pressure portal venous and hyperplasia and hypertrophy of lactotrophs in pregnancy cause the anterior pituitary to increase in size. Patients can have loss of all pituitary hormones or a selective loss, and the symptoms may only manifest at a later date. Empty-sella syndrome The pituitary gland is normally located within the sella turcica and empty-sella syndrome occurs in which the pituitary gland shrinks or becomes flattened meaning it cannot be seen on an MRI scan. The sella turcica is often filled with cerebrospinal fluid (CSF) which may have leaked, causing the pituitary gland to shrink or flatten. In most causes there are no symptoms or loss of pituitary function.

Kallmann’s syndrome

Kallmann’s syndrome is a rare genetic disorder resulting in a failure to start or complete puberty. Given this woman has managed to bear a child, this answer is incorrect. Pituitary apoplexy This is a result of bleeding from a pituitary gland tumour and presents with a sudden onset headache, diplopia and visual field defect due to compression of the optic chiasm. Subacute thyroiditis Subacute thyroiditis is usually associated with a goitre and presenting symptoms can include fever, myalgia, and malaise which are not described in the case history.

2394 Rate this question: 1 2 3 5 A 42-year-old woman is undergoing a magnetic resonance imaging (MRI) scan for neurological symptoms that are thought to represent early multiple sclerosis. The radiologist notes he is unable to visualise the pituitary gland and advises urgent clinical assessment. This reveals no symptoms of endocrine dysfunction and normal blood tests. What is the most likely cause for this appearance on an MRI scan? 1. Empty-sella syndrome 2. Kallmann’s syndrome 3. Pituitary apoplexy 4. Sheehan’s syndrome 5. Simmonds syndrome Explanation Empty-sella syndrome The pituitary gland is normally located within the sella turcica and empty-sella syndrome occurs in which the pituitary gland shrinks or becomes flattened meaning it cannot be seen on an MRI scan. The sella turcica is often filled with cerebrospinal fluid (CSF), which may have leaked in causing the pituitary gland to shrink or flatten. In most causes there are no symptoms or loss of pituitary function.

Kallmann’s syndrome

Kallmann’s syndrome is a rare genetic disorder resulting in a failure to start or complete puberty. Pituitary apoplexy Pituitary apoplexy is referred to as bleeding from a pituitary gland tumour which presents with a sudden onset headache, diplopia and visual field defect due to compression of the optic chiasm.

Sheehan’s syndrome

Sheehan’s syndrome is also known as Simmonds’ syndrome or post-partum pituitary gland necrosis. It is now a rare condition which occurs following pituitary infarction due to severe post-partum haemorrhage. The pituitary gland is vulnerable as it relies on a low pressure portal venous and hyperplasia and hypertrophy of lactotrophs in pregnancy cause the anterior pituitary to increase in size. Patients can have loss of all pituitary hormones or a selective loss, and the symptoms may only manifest at a later date. Simmonds syndrome As discussed, Simmonds’ syndrome is also known as Sheehan’s syndrome.

2395 Rate this question: 1 2 3 4 A 45-year-old man presented with urinary retention and saddle paraesthesia. His symptoms began 12 h ago. He has a cardiac pacemaker (of unknown make) in situ. Examination revealed reduced anal tone. What is the next step in his management?

1. CT myelogram of the lumbar spine 2. High dose corticosteroids 3. Immediate surgical decompression of the cervical spine 4. MRI scan of the lumbar spine 5. Physiotherapy, analgesia, and out-patient follow-up

Explanation

CT myelogram of the lumbar spine

This patient has classic symptoms of cauda equina syndrome. The most important next step is to identify the source of compression in the lumbar spine. The study of choice is a magnetic resonance imaging (MRI) scan. However in patients who are unable to undergo an MRI scan, such as those with non MRI safe pacemakers, a CT myelogram should be performed. Cauda equina syndrome is a complex of low back pain, bilateral or unilateral sciatica, saddle anaesthesia, motor weakness and bowel/bladder dysfunction. High dose corticosteroids Cauda equina is a surgical emergency. Decompression of the lumbar spine should be performed within 48 h from the onset of symptoms. Immediate surgical decompression of the cervical spine

Cauda equina syndrome affects the cauda equina which can be found in the lumbar spinal column not the cervical spine.

MRI scan of the lumbar spine

While some cardiac pacemakers are now MRI safe, given that the make of this man’s pacemaker is unknown, an MRI is contraindicated and he should proceed instead to a CT myelogram.

Physiotherapy, analgesia, and out-patient follow-up

Cauda equina is a surgical emergency. Decompression of the lumbar spine should be performed within 48 h from the onset of symptoms.

2428 Rate this question: 1 2 3 4 A 40-year-old man is involved in a road traffic collision and sustains a neck injury with C4/C5dislocation. On examination he has loss of motor function on one side and loss of pain and temperature sensation on the contralateral side. Which spinal cord syndrome is associated with these findings? 1. Anterior cord syndrome 2. Brown–Séquard syndrome 3. Central cord syndrome 4. Neurological shock 5. Posterior cord syndrome

Explanation

Brown–Séquard syndrome

Brown–Séquard syndrome is caused by complete cord hemitransection. On examination patients have loss of motor power, proprioception and vibration sensation on one side and contralateral loss of pain and temperature sensation. This syndrome has the best prognosis of all spinal cord injuries. Anterior cord syndrome Anterior cord syndrome occurs when there is loss of the anterior cord resulting in complete loss of motor function and pain and temperature sensation below the level of the injury, however, proprioception, vibration and fine touch will be preserved as these are carried in the posterior cord. Central cord syndrome

Central cord syndrome is one of the incomplete spinal cord injuries. Patients are often elderly with pre-existing spinal degeneration. The patient usually complains of loss of dexterity and hand weakness more pronounced than at shoulder and lower limbs. Prognosis is usually good with patients ambulatory at final visit but some loss of manual dexterity.

Neurological shock

This occurs when there is complete disruption of the cord usually at the cervical or high thoracic level resulting in loss of sympathetic tones, subsequently there is hypotension and bradycardia due to peripheral vasodilation. This is not described in the case history above. Posterior cord syndrome Posterior cord syndrome is very rare and occurs when the posterior cord is damaged, usually through ischaemia. Clinically this presents as loss of proprioception, vibration sense and light touch. This is not described in the case history above.

2430 Rate this question: A 65-year-old man presented with bilateral buttock and leg pain, worse with standing and walking, and better with leaning forward. On examination, he has normal power in his legs, straight leg raising and pedal pulses are normal. What is this clinical presentation consistent with? 1. Cauda equina syndrome 2. Disc prolapse 3. Osteomyelitis of the spine 4. Peripheral vascular disease 5. Spinal stenosis Explanation Spinal stenosis Spinal stenosis is due to narrowing of the spinal canal by either bony or soft tissue structures. Patients present with neurogenic claudication. MRI is the best investigation. Treatment is with analgesia and steroid injection, or surgical with decompression. It is important to differentiate between neurogenic and vascular claudication. Cauda equina syndrome

Cauda equina syndrome is a complex of low back pain, bilateral or unilateral sciatica, saddle anaesthesia, motor weakness (note this patient has normal power in his legs) and bowel/bladder dysfunction. Disc prolapse Lumbar disc prolapse is a condition occurring in fit young adults. It occurs acutely when lifting a heavy weight or while straining. The pain is felt in the lower back, sometimes radiating to the buttock or down the leg. In total, 90% of patients will have improvement of symptoms within 3 months with non-operative care. MRI is the modality of choice for diagnosis of lumbar disc herniations. Osteomyelitis of the spine This is a bony infection of the spine and, as this patient has no signs of infection, this is unlikely. Peripheral vascular disease Pedal pulses are normal in this patient and makes peripheral vascular disease unlikely as a cause for his symptoms.

2432 Rate this question: 1 2 3 4 5

Extradural bleeding is associated with severe head injury and can result in permanent brain damage if not recognised and managed immediately. The condition strips the dura mater from the periosteum of the skull. What is the main radiological feature of extradural haematoma?

1. Absence of a fracture on plain X-rays 2. Biconvex looking haematoma on computed tomography (CT) scan 3. Bleeding into the ventricles on CT 4. Bleeding that crosses suture lines 5. Crescentic shape of the bleed on CT Explanation

Biconvex looking haematoma on computed tomography (CT) scan

Extradural haematomas are biconvex and do not cross suture lines, as the dura is very adherent to the cranium. They are usually associated with a fracture of the pterion, which is formed by the meeting of the frontal, parietal, temporal, sphenoid bones (located just posterior to the temple). This is the weakest part of the cranium and the anterior branch of the middle meningeal artery runs just beneath the pterion and so is often damaged (causing an extradural haematoma) with trauma affecting this area of the skill. Both extradural and subdural haematomas and any condition leading to cerebral oedema could lead to a rise in ICP. In the early stages of head injury, there is a non-linear relationship between an expanding haematoma and elevation of ICP – a haematoma may expand without any significant rise in pressure. Once this early compliance is lost, the pressure will rise rapidly. This severely jeopardises cerebral perfusion: cerebral perfusion pressure (CPP) is equal to mean arterial blood pressure minus ICP (normal CPP is approximately 70 mmHg and the normal ICP in adults is 10– 15 mmHg). The Monro–Kellie hypothesis: the sum of intracranial volumes of blood, brain, cerebrospinal fluid (CSF) and other components is constant, an increase in any one of these must be offset by a corresponding decrease in another, or else the ICP will rise.

Absence of a fracture on plain X-rays

Extradural haematomas are usually associated with skull fractures. Bleeding into the ventricles on CT Bleeding occurs between the dura and skull. Bleeding into the ventricles would suggest a sub-arachnoid bleed. Bleeding that crosses suture lines Extradural haematomas do not cross suture lines due to the adherent nature of the dura mata to the cranium. Crescentic shape of the bleed on CT

Subdural haematoma are crescentic in shape.

2442 A 20-year-old man sustains a head injury while skiing without protective head gear. Initiallyhe showed signs of concussion, but recovered and did not seek medical attention. 2 days later he is brought to the emergency department with a GCS of 6. What is the most likely cause? 1. Cerebral contusion 2. Diffuse axonal injury 3. Extradural haematoma 4. Intracerebral haematoma 5. Subdural haematoma

Explanation Extradural haematoma There are a number of different mechanisms of brain injury including hypoxia, ischaemia, contusion and diffuse axonal injury. Focal injuries include extradural, subdural and intracerebral haematomas and contusions. Extradural haematomas often result from a trivial blow, and children and young adults are more at risk. Typically, initially there are signs of concussion, then a lucid interval, then rapid decompensation as intracranial pressure rises. Cerebral contusion Cerebral contusions can have varying presentations depending on location and severity. The risk is off raised intracranial pressure due to cerebral swelling. It is unlikely to present with reduced conscious level 2 days post injury, and more likely to present in the first 12 h following injury. Diffuse axonal injury Diffuse axonal injury will present immediately following the incident and is a result of extensive lesions in the white matter tracts. Intracerebral haematoma An intracerebral haematoma would likely present immediately with a focal neurological deficit following injury due to the disruption of the cerebral cortex. Subdural haematoma Subdural haematomas are more common in the elderly due to the fragility of the bridging veins, so it would be unlikely in a 20-year-old. There are a number of different mechanisms of brain injury including hypoxia, ischaemia, contusion and diffuse axonal injury. Focal injuries include extradural, sudural and intracerebral haematomas and contusions. Extradural haematomas often result from a trivial blow, and children and young adults are more at risk. Typically, initially there are signs of concussion, then a lucid interval, then rapid decompensation as intracranial pressure rises.

2447 Rate this question: 1 2 A 45-year-old man presents to the ENT clinic. On audiology it is apparent that he has unilateral sensorineural deafness and tinnitus. What intracranial lesion is most likely to cause this? 1. Astrocytoma 2. Ependymoma 3. Medulloblastoma 4. Oligodendroma 5. Schwannoma Explanation Schwannoma Schwannoma, otherwise known as acoustic neuroma, usually arise from cranial nerve VIII. The diagnosis of schwannoma must be considered in all patients presenting with a unilateral sensorineural deafness. Astrocytoma This is the most common form of brain neoplasm and is a subtype of gliomas. It would not cause sensorineural hearing loss. Ependymoma This is a subtype of gliomas and it would not cause sensorineural hearing loss. Medulloblastoma Again, this is a subtype of gliomas and it would not cause sensorineural hearing loss. Oligodendroma This is a type of glioma which is relative slow growing but survival rates are poor. It would not cause sensorineural hearing loss.

2455 Rate this question: 1 2 3 4 5

A 34-year-old man is admitted with an ulnar-sided wrist incised wound having fallen onto glass while intoxicated. He is no longer able to move his hand normally. Which of the following is a sign of ulnar nerve transection? 1. Loss of flexor digiti minimi function 2. Loss of the index finger flexor digitorum 3. Loss of sensation in the radial 3 digits 4. Loss of sensation over the 1st dorsal web space 5. Loss of thumb abduction

Explanation Loss of flexor digiti minimi function The ulnar nerve supplies flexor carpi ulnaris (FCU) and medial half of flexor digitorum profundus in the forearm. In the hand it gives off a superficial and deep branch. The deep branch supplies the hypothenar muscles, opponens digiti minimi, abductor digiti minimi and flexor digiti minimi as well as the third and fourth lumbrical muscles, dorsal and palmar interossei, adductor pollicis and deep head of flexor pollicis brevis. The superficial branch of the ulnar nerve supplies palmaris brevis.

Loss of the index finger flexor digitorum

The ulnar nerve only supplies the medial half of flexor digitorum profundus function and the index finger would represent the lateral half. Loss of sensation in the radial 3 digits

Sensation in the radial 3 digits would be supplied by the median nerve. Loss of sensation over the 1st dorsal web space This area is supplied by the radial nerve. Loss of thumb abduction

Thumb abduction by abductor pollicis brevis is inserted by the recurrent branch of the median nerve.

2656 Rate this question: 1 2 3 4 5

A previously well 61-year-old man presents to the Emergency Department with a marked right upper limb weakness and right facial droop. He is also noted to have a dysphasia. On taking a history from his wife it emerges he is due to attend hospital the following day for an elective inguinal hernia repair. He is normally on warfarin for a mechanical valve, which he stopped 6 days ago on the advice of his GP. What complication has occurred? 1. Amaurosis fugax 2. Embolic stroke 3. Haemorrhagic stroke 4. Prosthetic endocarditis 5. Subdural haematoma Explanation Embolic stroke

Mechanical valvular prostheses require lifelong anticoagulation to prevent emboli and valvular obstruction with thrombus. Heparin may be used to anticoagulate should warfarin need to be withheld for any reason. Warfarin is a teratogen, and so mechanical valves should be avoided in women who plan to have children. Amaurosis fugax Amaurosis fugax is the painless loss of vision in one or both eyes, this is not the case described above. Haemorrhagic stroke

The patient has stopped taking warfarin so reducing his risk of a haemorrhagic stroke. Prosthetic endocarditis

Prosthetic valve endocarditis is notoriously difficult to treat and will often require redo valve replacement. So antibiotic prophylaxis before instrumentation is very important in those patients with prosthetic valves. While this may lead to abscess formation in the brain due to seeding from the valve endocarditis, the case history is of a stroke. Subdural haematoma

This usually occurs following trauma of which there is none described in this case history.

2870 Rate this question: 1 2 3 4 5

You are examining a newborn baby, who was suspected to have anomalies on prenatal screening. You note a sac protruding from the lumbosacral region of the baby’s back. The spinal cord is unfolded within. What associated condition is most commonly seen in children with this? 1. Biliary atresia 2. Cleft palate 3. Down syndrome 4. Gastroschisis 5. Hydrocephalus Explanation Hydrocephalus The baby has a myelomeningocele, which is a neural tube defect. There is complete unfolding of the cord, which lies on the childs back, most commonly at the lumbosacral junction. 80% of patients with myelomeningocele also have hydrocephalus. Biliary atresia

Biliary atresia describes malformation of the biliary tree in which some bile ducts may be blocked, stenosed or absent. It does not have an association with myelomeningoceles. Cleft palate

Cleft palate is most commonly associated with Pierre Robin sequence (PRS). This is characterised by micrognathia, retraction of the tongue and upper airways obstruction. Down syndrome

Down’s syndrome is a chromosomal abnormality as a result of trisomy 21. While there is an association between neural tube defects and Down’s syndrome, hydrocephalus is more common. Gastroschisis

Gastroschisis occurs when the small intestine fails to return to the abdominal cavity, as such the neonate is born with a section of bowel protruding through the abdominal wall. It is not associated with neural tube defects.

3091 Rate this question: 1 2 3 4 5

On examining a newborn you notice a fluid-filled meningeal sac filled with cerebrospinal fluid (CSF), but no evidence of the cord, over the lower lumbar region. The mother is very anxious about her child and wants your advice. What condition does the patient have? 1. Cerebral palsy 2. Hydrocephalus 3. Meningocele 4. Myelomeningocele 5. Spina bifida occulta Explanation Meningocele

The incidence of neural tube defects has decreased over the last 30 years. These defects are broadly classified into three types: myelomeningocele, meningocele and spina bifida occulta. A fluid filled sac without the cord contained is a meningocele. Cerebral palsy

Cerebral palsy is a broad term which refers to permanent movement disorders due to abnormal development of the brain. It usually presents in early childhood with movement difficulties, not with a meningeal sac. Hydrocephalus

Hydrocephalus is a condition in which there is excess accumulation of cerebrospinal fluid in the brain causing dilatation of the ventricular system and increased intracranial pressure. It does not present with a meningeal sac. Myelomeningocele

The presence of the cord within the sac would classify as a myelomeningocele. Spina bifida occulta In spina bifida occulta there is a covering of skin and no fluid filled sac.

3092 Rate this question: 1 2 3 4 5

A pregnant woman has increased levels of serum and amniotic fluid alpha-fetoprotein. Ultrasound scan (USS) reveals that the neural tube has failed to close rostrally. What condition will the baby have? 1. Anencephaly 2. Exomphalos 3. Meningocele 4. Myelomeningocele 5. Spina bifida occulta Explanation Anencephaly Neural tube defects can be detected pre-natally. The neural tube closure begins centrally and extends rostrally on day 24 and caudally on day 26 of gestation. Failure to close rostrally results in anencephaly. Alpha-fetoprotein levels can be raised in a number of conditions, and forms part of Down’s syndrome screening, and screening for neural tube and abdominal wall defects. Exomphalos Exomphalos is a condition in which the primitive bowel loop fails to return to the abdominal cavity. Meningocele A fluid-filled meningeal sac without the cord contained is a meningocele.

Myelomeningocele

The presence of a meningeal sac and spinal cord protruding through an opening would classify as a myelomeningocele. Spina bifida occulta Failure of the neural tube to close caudally results in spina bifida, which can result in three distinct anomalies; myelomeningocele, meningocele, spina bifida occulta. In spina bifida occulta there is a covering of skin and no fluid filled meningeal sac.

3096 Rate this question: 1 2 3 4 5

You are treating a neonate born with a myelomeningocele. You were not expecting this on the baby check and perform a more thorough examination and investigations to check for any additional congenital defects. What is the commonest associated condition that you should check for? 1. Anencephaly 2. Exomphalos 3. Hip dislocation 4. Hydrocephalus 5. Syringomyelia Explanation Hydrocephalus In myelomeningocele there is complete unfolding of the spinal cord. It is more common than meningocele. There are a number of conditions associated with it including syringomyelia and hip dislocation, but hydrocephalus is the most common, seen in 80– 90% of those with myelomeningocele. Anencephaly Anencephaly can be part of rachischisis, the most severe form of spina bifida cystica, but is not associated with myelomeningocele. Exomphalos Exomphalos is a condition in which the primitive bowel loop fails to return to the abdominal cavity, and does not appear to have an association with myelomeningoceles. Hip dislocation Myelomeningocele in association with hip dislocation is not as common as it’s association with hydrocephalus. Syringomyelia Syringomyelia is a broad term used to refer to formation of a cavity or cyst within the spinal cord, and while it can be associated with a myelomeningocele, hydrocephalus is a more common association.

3103 Rate this question: 1 2 3 4 5

A 45-year-old man was referred by his GP with back pain, urinary retention and saddle paraesthesia for 10 h. On examination, he has bilateral altered sensation in his legs and reduced anal tone. What is the next step in management? 1. Caudal epidural steroid injection 2. Computed tomography (CT) myelogram of the lumbar spine 3. Immediate surgical decompression of the cervical spine 4. MRI scan of the lumbar spine 5. Physiotherapy, analgesia, and out-patient follow up Explanation

MRI scan of the lumbar spine

This patient has classic symptoms of cauda equina syndrome. The most important next step is to confirm the diagnosis with an MRI scan to evaluate neurological compression. In patients who are unable to undergo an MRI such as those with pacemakers and claustrophobia, a CT myelogram is an alternative. Cauda equina syndrome is a complex of low back pain, bilateral or unilateral sciatica, saddle anaesthesia, motor weakness and bowel/bladder dysfunction. Studies have shown improved outcomes in bowel and bladder function and resolution of motor and sensory deficits when decompression performed within 48 h of the onset of symptoms. Caudal epidural steroid injection This would not be helpful in this case, he requires imaging to assess for cauda equina syndrome and then urgent decompression.

Computed tomography (CT) myelogram of the lumbar spine

CT myelogram of the lumbar spine is an alternative choice for those unable to undergo a magnetic resonance imaging (MRI) scan, however, MRI is the investigation of choice. Immediate surgical decompression of the cervical spine If the patient had compression of the spinal cord at the cervical spine the presenting signs and symptoms would be very different; upper limb neurological deficit would be expected. However, this man clearly has evidence of compression of the cauda equina in the lumbar spine.

Physiotherapy, analgesia, and out-patient follow up

The patient has multiple ‘red flags’ for acute cauda equina syndrome and requires immediate investigation with an MRI scan. This course of action would therefore be inappropriate and associated with high morbidity.

3236 Rate this question: 1 2 3 A 35-year-old woman presents with an acute episode of low backache which started on lifting a heavy object at home. Her pain is radiating down one leg. What is the most likely diagnosis? 1. Facet dislocation 2. Lumbar disc herniation 3. Multiple myeloma 4. Spinal stenosis 5. Spondylolisthesis Explanation Lumbar disc herniation

Lumbar disc prolapse is a condition occurring in fit young adults. It occurs acutely when lifting a heavy weight or while straining. The pain is felt in the lower back, sometimes radiating to the buttock or down the leg. For 90% of patients, improvement of symptoms will have occurred within 3 months with non-operative care. MRI is the modality of choice for diagnosis of lumbar disc herniations. Facet dislocation

Facet dislocation can occur with or without the presence of an associated fracture, however, it usually occurs due to forced flexion of the cervical spine as this is the most mobile section of the spinal column. Given this patient has lower back pain, facet dislocation is unlikely. Multiple myeloma

Multiple myeloma is a neoplastic haematological condition most commonly seen in the elderly which often presents with bone pain (often affecting the spine) due to presence of lytic lesions within the bone. It would be rare in a woman of this age. Spinal stenosis

Spinal stenosis is an abnormal narrowing of the spinal canal and is most commonly seen in the elderly in the context of arthritis. Spondylolisthesis Spondylolisthesis is the displacement of one vertebrae in relation to another, and has a number of aetiologies, including trauma and degenerative. It would be uncommon given the case history.

3241 Rate this question: 1 2 3 4 5

Following a road traffic collision, a patient presents with loss of motor function in the upper limbs and less so on the lower limbs. The patient complains of a loss of dexterity in his hands. Which spinal cord syndrome is associated with these findings?

1. Anterior cord syndrome 2. Brown–Séquard syndrome 3. Central cord syndrome 4. Neurological shock 5. Posterior cord syndrome Explanation Central cord syndrome Central cord syndrome is one of the incomplete spinal cord injuries. Patients are often elderly with pre-existing spinal degeneration. The patient usually complains of loss of dexterity and hand weakness more pronounced than at shoulder and lower limbs. Prognosis is usually good with patients' ambulatory at final visit but some loss of manual dexterity.

Anterior cord syndrome

Anterior cord syndrome occurs when there is loss of the anterior cord resulting in complete loss of motor function and pain and temperature sensation below the level of the injury, however, proprioception, vibration and fine touch will be preserved as these are carried in the posterior cord. The loss of dexterity is more indicative of central cord syndrome.

Brown–Séquard syndrome

Brown–Séquard syndrome occurs when there is hemisection of the spinal cord. It results in loss of motor function and proprioception on the ipsilateral side, while loss of pain and temperature sensation of the contralateral side of the injury. This scenario is not described in the case above.

Neurological shock

This event occurs when there is complete disruption of the cord usually at the cervical or high thoracic level resulting in loss of sympathetic tones, subsequently there is hypotension and bradycardia due to peripheral vasodilation. This scenario is not described in the case history above. Posterior cord syndrome

Posterior cord syndrome is very rare and occurs when the posterior cord is damaged, usually through ischaemia. Clinically this presents as loss of proprioception, vibration sense and light touch. This is not described in the case history above.

3244